Vous êtes sur la page 1sur 28

Solutions to Chapter 5 Problems

5-1

No. A higher MARR reduces the present worth of future cash inflows created by
savings (reductions) in annual operating costs. The initial investment (at time 0)
is unaffected, so higher MARRs reduce the price that a company should be
willing to pay for this equipment.

5-2

Monthly gasoline savings = 10 gallons x $2.00 per gallon = $20 per month.
$1,200 = $20 per month x (P/A, 0.5% per month, N months);
N = 72 months to recover the investment plus interest

5-3

(a)

PW (12%) = - $640,000 + $180,000 (P/A,12%,8) - $42,000 (P/A,12%,8)


+ $4,000 (P/G, 12%,6)(P/F,12%,2) + $20,000 (P/F,12%,8)
= - $640,000 + $180,000 (4.9676) - $42,000 (4.9676)
+ $4,000 (8.930)(0.7972) + $20,000 (0.4039)
= $82,082.78 > 0, therefore the proposal is acceptable

(b) Based on PW and IRR, the proposal is acceptable.


5-4

PW(12%) = -$13,000 + $3,000 (P/F,12%,15) - $100 (P/A,12%,15)


- $200 (P/F,12%,5) - $550 (P/F,12%,10)
= -$13,000 + $3,000(0.1827) - $100(6.8109) - $200(0.5674)
- $550(0.3220)
= -$13,423.57

5-5

PW(15%) = -$10,000 + ($8,000 - $4000)(P/A,15%,5) - $1,000 (P/F,15%,5)


= -$10,000 + $4000(3.3522) - $1,000 (0.4972)
= $2,911.60
FW(15%) = -$10,000 (F/P,15%,5) + ($8,000 - $4000)(F/A,15%,5) - $1,000
= -$10,000 (2.0114) + $4000(6.7424) - $1,000
= $5,855.60
AW(15%) = $8,000 - $4000 - [$10,000(A/P,15%,5) - (- $1,000)(A/F,15%,5)]
= $4000 - [$10,000(0.2983) + $1,000(0.1483)]
= $868.70

5-6

Assume miles driven each year is roughly constant at 12,000. A four-speed


transmission car will consume 400 gallons of gasoline each year, and the fuel cost
is $800 per year. The annual savings due to a six speed transmission is 0.04 ($800) =
$32. Over a ten year life, the present worth of savings will be $32 (P/A, 6%, 10) =
$235.52. This is how much extra the motorist should be willing to pay for a sixspeed transmission.

101

Chapter 5 Solutions

5-7

Your Point of View:

$1,300

$1,400 $1,500

$1,800
$1,600 $1,700
+

$12k

EOY

$10,000
(R-E) = $1,300 (yr. 1)
= $1,300 + (k-1)($100) for 1 k 6

5-8

PW(15%) = -$10,000 + $1,300 (P/A, 15%, 6) + $100 (P/G, 15%, 6) +


$1,200 (P/F, 15%, 6)
= -$10,000 + $1,300 (3.7845) + $100 (7.937) + $12,000 (0.4323)
= $901.15 > 0; the investment appears to be a good one if the risk
is low
Capital Investment = -$300,000 - $600,000 - $250,000 - $100,000 = -$1,250,000
Revenue = $750,000 per year
Market Value = $400,000 + $350,000 + $50,000 = $800,000
Expenses = -$475,000 per year
PW(15%) = -$1,250,000 + ($750,000 - $475,000)(P/A,15%,10)
+ $900,000(P/F,15%,10)
= -$1,250,000 + $275,000(5.0188) + $800,000(0.2472)
= $327,930 > 0
Therefore, they should invest in the new product line.

5-9

Both cars last for 6 years. Cost of gasoline stays constant. Assume that fuel
expense is the only expense to consider (ignore tire wear, etc.). Assume a
standard auto gets 25 mpg.
Composite auto annual fuel expense
(20,000 mi./yr.)(1 gal./75 mi.)($2.00/gal.) = $533.33/yr.

Chapter 5 Solutions

102

Traditional auto annual fuel expense


(20,000 mi./yr.)(1 gal/.25 mi.)($2.00 gal.) = $1,660/yr.
So spending an extra P = ($1,600 - $533.33)(P/A, 10%, 6) will be justified by the
annual fuel savings.
P = $1,066.67 (4.3553) = $4,645.67
If the composite auto costs more than $4,646 compared to the traditional auto, the
purchase should not be made. (This ignores prestige value of being the first on the
block with a composite car.)
Reduced fuel expense per year is being traded off against extra investment for the
composite auto.
5-10 Desired yield per 6 months = 3%; N = 3 x 2 = 6 periods
i = [(1.13)1/2 1)]100 = 6.3% per semi-annual period
VN = $1,000 (P/F, 6.3%,6) + 0.03 ($1,000) (P/A,6.3%,6)
= $1,000 (0.693) + $30 (4.876)
= $69.30 + $146.28 = $836.58
5-11 Desired yield per year = 10%
VN = $1,000 (P/F, 10%, 10) + 0.14 ($1,000) (P/A,10%,10)
= $1,000 (.3855) + $140 (6.1446) = $1,245.74
5-12

Desired yield per quarter = 12%/4 = 3%; N = 4(10) = 40 quarters


(a)

VN = $10,000 (P/F,3%,40) + 0.02 ($10,000) (P/A,3%,40)


= $10,000 (0.3066) + $200 (23.1148)
= $7,688.96

(b)

Quarterly interest payment = (0.02)($1,000,000) = $20,000

0.08
i/qtr = 1 +

1/ 2

1 = 0.0198 or 1.98%

Equivalent value of interest payments at EOY10:


F40 = $20,000(F/A,1.98%,40) = $20,000(60.1408) = $1,202,816
F40 redemption cost = $1,000,000
2

0.08
i/yr = 1 +
1 = 0.0816 or 8.16%

103

Chapter 5 Solutions

A = [$1,000,000 + $1,202,816] (A/F,8.16%,10)


= $2,202,816 (0.0685)
= $150,892.90
5-13 True interest rate in the IRR associated with the equation PW(i') = 0.
($1,000,000 $50,000) $40,000 (P/A,i',15%) - $70,256 (P/A,i',15)
$1,000,000 (P/F,i',15) = 0
$950,000 - $110,526 (P/A,i',15%) - $1,000,000 (P/F,i',15) = 0
at 10% $950,000 - $110256 (7.6061) - $1,000,000 (0.2394) = -$128,018
at 12% $950,000 - $110,256 (6.8109) - $1,000,000 (0.1827) = +$16,357
by interpolation, i' = 11.773%
5-14

Interest payments from bond = $100,000 (0.0725) = $7,250 every six months
P0 = 0 = -$100,000 + $7,250 [P/A,i'%,10(2)] + $110,000 [P/F,i'%,10(2)]
PW(7%) = $5,230.50 > 0, i'% > 7%
PW(8%) = -$5,223.78 < 0, i'% < 8%
Linear interpolation between 7% and 8% yields: i'% = 7.5% per six months.

5-15

Original Mortgage Payment: i / mo = 10%/12 = 0.833%


A = $100,000(A/P,0.833%,360)
A = $100,000(0.0087757) = $877.57
Remaining principal after two years of payments:
P24 = $877.57(P/A,0.833%,336)
P24 = $877.57(112.61763) = $98,829.86
New payment if refinanced over remaining 28 years:
0.583%

i / mo = 7%/12 =

A = $98,829.86(A/P,0.583%,336)
A = $98,829.86(0.0067961) = $671.66
Savings per month if refinanced = $877.57 - $671.66 = $205.91
Number of months Susie should remain in her townhouse to make this a
worthwhile venture:
$4,500 = $205.91(P/A,0.5%,N)
(P/A,0.5%,N) = $4,500/$205.91 = 21.8542
From Table C-2, N 24 months.

Chapter 5 Solutions

104

5-16

PW

10

11

2000
2600
3200
4100

4700

5000

PW of the base ($2,000) = $2,000 (P/A,10%,11) = $2,000 (P/A,10%,11)


(1.10)11 1
= $2,000
= $12,990.12
11
(0.10)(1.10)
PW of the gradient ($300) = $300 (P/G, 10%,11) =

1
300 (1.10)11 1
11

11
0.10 0.10
(1.10)

= $7,818.88
Total PW = $12,990.12 + $7,818.88 = $20,909.00
5-17 PW of 5 yr lining:
PW = [ $2,500 (A/P, 10%,5)] (P/A,10%,20)
= $2,500 (0.2638)(8.5136) = $5,614.72

PW 10 yr lining:
PW = [$6,500 (A/P,10%,10)] (P/A,10%,20)
= $6,500 (0.1627) (8.5136) = $9,003.56

5-18
Principal contributed
Future worth at 6% per year
a
b

Janet
$10,000
$56,571a

Bill
$25,000
$54,865b

$1,000 (F/A,6%,10) (F/P,6%,25)


$1,000 (F/A,6%,25)
Janet will have accumulated more money in her retirement account than Bill by
age 65 even though her contribution of principal was less than half of what Bill
invested. The reason for this is that Janet had compounding of interest working
for her longer than Bill had. The lesson is start saving early!

105

Chapter 5 Solutions

5-19
$2,900 $2,900 $2,800 $2,700 $2,500 $2,400 $2,300 $2,200 $2,100

$500

2 3 4 5 6 7 8 9 10

(EOY)

Let A = $2,900, G = -$100 (delayed 1 year)


F6 = -$2,000
P0 = $2,900 (P/A,6$,10) 100(P/G,6%,9)(P/F,6%,1) - $2,000(P/F,6%,6)
= $2,900 (7.3601) - $100 (24.5768) (0.9434) - $2,000 (0.7050)
= $17,615.71
5-20

0.1 P = P (A/P,8%,N) so N 21 years


A payout duration table can be constructed for selected payout percentages and
compound interest rates as follows:
Interest Rate/Year
4%
6%
8%
10%
13.0*
15.7
20.9
never
Payout/Yr 10%
(% of principal) 20%
5.7
6.1
6.6
7.3
30%
3.7
3.8
4.0
4.3
* Note: Table entries are years.

5-21

F14 = A(F/A,4%,17) = 23.6975A


F11 = 23.6975A (P/F,4%,3) = 21.067A
so then 21.067A = $32,500(P/F,4%,1)+ $34,125(P/F,4%,2) + $35,750 (P/F,4%,3)
and A = $4,490/year

5-22

(a) CW(10%) =

$1,500 $10,000 (A F ,10%,4)


+
( P / F,10%,1) = $34,591
010
.
010
.

(b) Find the value for N for which (A/P,10%,N) = 0.10


From Table C-13, N = 80 years
5-23

Opportunity Cost = Investment at BOY x (P/F, 15%, 1) = BOY (0.15)


Capital Recovery Amount = Opportunity Cost + Loss in Value During Year

Chapter 5 Solutions

106

Investment
Opportunity
at Beginning
Cost of Interest
of Year
(i = 15%)
Year
1
$10,000
$10,000 (0.15) = 1,500
2
7,000 (0.15) = 1,050
10,000-3000 = 7,000
3
5,000 (0.15) = 750
7,000 - 2,000 = 5,000
4
3,000 (0.15) = 450
5,000 - 2,000 = 3,000

Loss in
Capital
Value
Recovery
During Year
Amount
$3,000
1,500 + 3,000 = 4,500
2,000
1,050 + 2,000 = 3,050
2,000
750 + 2,000 = 2,750
1,000
450 + 1,000 = 1,450

P0 = $4,500 (P/F,15%,1) + $3,050 (P/F,15%,2) + $2,750 (P/F,15%,3)


+$1,450 (P/F, 15%, 4)
= $4,500 (0.8696) + $3,050 (0.7561) + $2,750 (0.6575) + $1,450 (0.5718)
= $ 8,856.54
A = $8,856.54 (A/P,15%,4) = $8,856.49 (0.3503) = $3,102.45
This same value can be obtained and confirmed with Equation 5-5:
CR(i%) = I (A/P, i%, N) - S (A/F, i%, N)
= $10,000 (A/P,15%,4) - $2,000 (A/F,15%,4)
= $10,000 (0.3503) - $2,000 (0.2003)
= $3,102.12
Note: The Annual Worth from the table and the CR amount from Equation 5-5
are the same.
5-24

FW(18%) = -$8,000 (F/P,18%,10) + $2,400 (F/A,18%,10)


= -$8,000 (5.2338) + $2,400(23.5213)
= $14,580.72
Therefore, recommend process R since FW 0.
AW(18%) = -$8,000 (A/P,18%,10) + $2,400
= -$8,000 (0.2225) + $2,400
= $620
Therefore, recommend process R since AW 0.

5-25 AW (of costs) = $125,000 (A/P,8%,10) + $1,000 + $1,000 (A/G,8%,10)


$40,000 (A/F,8%,10)
= $125,000 (0.14900) + $1,000 + $1,000 (3.8713) - $40,000 (0.0690)
= $18,625 + $1,000 + $3,871.30 - $2,760 = $20,736.30 per year

107

Chapter 5 Solutions

5-26

Year
1
2
3
4

5-27

Investment
Opportunity
at Beginning
Cost of Interest
of Year
(i = 15%)
$1,000
$50
1,000 - 200 = 800 800(0.05) = 40
600
30
600 - 200 = 400
20

Loss in
Value
During Year
$250 - $50 = $200
200
200
400 - 300 = 100

Capital
Recovery
Amount
$250
240
230
20 + 100 = 120

(a)
(b)
(c)
(d)
(e)
(f)

Loss in Value = Capital Recovery Amount - Opportunity Cost


Investment at BOY2 = Investment at BOY1 - Loss in Value during Year 1
Opportunity Cost = Investment at BOY * (0.05)
Investment at BOY4 = Investment at BOY3 - Loss in Value during Year 3
Loss in Value during year 4 = Investment at BOY4 - Salvage Value at EOY4
Capital Recovery Amount = Opportunity Cost + Loss in Value during Year

(a)

1
lim (P / F,i%, N) = lim
=0
i
i (1 + i) N

Therefore, as i , the PW approaches -$3,000.


(b)

By inspection, = 5 years. Therefore ' 5 years


PW0-5(12%) = -$3,000 - $1,000(P/F,12%,1) + $1,000(P/A,12%,3)(P/F,12%,1)
+ $2,000(P/F,12%,5)
= -$3,000 - $1,000(0.8929) + $1,000(2.4018)(0.8929)
+ $2,000(0.5674)
= -$613.53 < 0 , ' > 5 years
PW0-6(12%) = PW0-5(12%) + $2,000(P/F,12%,6)
= -$613.53 + $2,000(0.5066)
= $399.67 > 0, ' = 6 years

(c)

PW(0%) = -$3,000 - $1,000(P/F,0%,1) + $1,000(P/A,0%,3)(P/F,0%,1)


+ $2,000(P/F,0%,5) - $2,000(P/F,0%,6)
= - $3,000 - $1,000(1) + $1,000(3)(1)+ $2,000(1) - $2,000(1)
= - $1,000
AW(0%) = PW(0%)(A/P,0%,6)
= - $1,000 (0.1667)
= - $166.70

Chapter 5 Solutions

108

5-28

(a)

(Maintenance, taxes, insurance) = $2,000 + 0.05 ($75,000) = $5,750/year


cV =

$1.00 / unit direct labor


+ $0.75 / unit new material
+ $ 0.30 / unit scrap
+ $ 0.60 / unit overhead
$ 2.65 / unit

X = annual production in units

[Unit costs (not including CR)] = $2.65 / unit +

$5,750
= $2.94 / unit
20,000 units

CR = ($75,000+$15,000)(A/P,20%,5) - ($3,000+$15,000)(A/F,20%,5)
= $90,000 (0.3344) - $18,000 (0.1344)
108
= $27,676.80
CR / unit =

$27,676.80
= $1.38 / unit
20,000 units

Total Cost / unit = CT = $2.94 + $1.38 = $4.32 / unit


Thus the unit price for the bid should be greater than $4.32.
(b)

Selling Price = (1 + 0.20) CT = (1.20) ($4.32 / unit) = $5.18 /unit

5-29

$10,000 = $200 (F/A,i,45) (F/P,i,3) or i per month is approximately equal to


0.4165% which equates to i per year of (1.004165)12 1 = 0.0511 (5.11% per
year). This is a conservative investment when Stan makes the assumption that the
investment firm will pay him $10,000 when he leaves the service at the end of 4
years (i.e., there is little risk involved). Stan should probably take this
opportunity
to invest money while he is in the service. It beats U.S. savnings bonds which
pay
about 4% per year.

5-30

PW(i'%) of outflows = PW(i'%) of inflows


$308.57 (P/A,i'%,35) = $7,800
(P/A,i'%,35) = 25.2779
(P/A,1%,35) = 29.4085 and (P/A,2%,35) = 24.9986, Therefore, 1% < i'% < 2%.
Linear interpolation yields: i'% = 1.9% per month

109

Chapter 5 Solutions

A.P.R = (1.9% per month)(12 months/year)


= 22.8% compounded monthly

5-31
$1,000

i=?

23

24

EOM

$56.44

$1,000 = 56.44 (P/A,i',24)


i
(P/A,i,24)
-----------------------------------2
18.9139
i'
17.7174
16.9355
3

i'2
1
=
18.9139 17.7174 18.9139 16.9355
i '2
1
=
1.1960 1.9784
i' 2 =

1.196
1.9784

i' = 2.604% / mo.; i/yr = (1.026)12 1 = 0.3607 or 36.1% / yr.


5-32

$1,000 = $58.50 (P/A,i'%,24)


(P/A,i'%,24) = $1,000/$58.50 = 17.094
From Appendix C: (P/A,2%,24) = 18.9139 and (P/A,3%,24) = 16.9355
By linear interpolation: i'% = 2.9% per month
Thus, ieffective = (1.029)12 - 1 = 0.409 or 40.9% per year.

5-33

PW = 0 = $1,000 - $117 (P/A,i'%,2) - $58.50(P/A,i'%,20) (P/F,i'%,2)


By trial and error: PW(3%) = -$44 and PW(4%) = $44
By linear interpolation: i'% = 3.5% per month
Thus, ieffective = (1.035)12 - 1 = 0.511 or 51.1% per year.

Chapter 5 Solutions

110

5-34

PWcost = PWbenefit
$17,000 = (0.8) ($4,000) (P/A,i%,10) + (0.8)($300)(P/G,i%,10)
i = 14%

5-35
Year
0
1

Plan 2
-$850
0

Plan 1
-$90
-$90

Plan 2 Plan 1
-$760
+$90

A = P x i (as a decimal)
i = A/P, = $90 / $760 = 11.84%
Choose Plan 1 if MARR > 11.84%
5-36

Assume that the task is to produce 40,000 pieces / yr.


Old Machine (No Brake):
Production time/year = (40,000 pieces/yr)(2.5 min/pc)(1hr/60 min)
= 1,666.67 hr/ yr
Operator salary = $16.50/hr (1,666.67 hr / yr)
= $27,500.06 / yr
Overhead = $4.00/hr (1,666.67 hr/ yr)
= $ 6,666.68 / yr
Total Cost
= $34,166.74
New Machine (With Brake):
Production time/year = (40,000 pieces/yr)(2.05 min/pc)(1hr/60 min)
= 1,366.67 hr/yr
Operator salary = $16.50 / hr( 1,366.67 hr / yr)
= $22,550.06 / yr
Overhead = $4.00 / hr (1,366.67 hr/ yr)
= $ 5,466.68 / yr
Maintenance
= $ 250.00 / yr
Total Cost
= $28,266.74/ yr
Savings per year (with brake) = $34,166.74 - $28,266.74 = $5,900 / yr
Thus, PW = 0 = X + $5,900 (P/A,15%,5)
X = - $5,900 (3.3522)
X = - $19,778
Thus the company could afford to pay up to $19,778 for a new brake.

5-37

General Equation:
PW(i'%) = 0 = -$450,000 - $42,500(P/F,i'%,1) + $92,800(P/F,i'%,2)
+ $386,000(P/F, i'%, 3) + $614,600(P/F, i'%, 4)
- $202,200(P/F, i'%, 5)
PW(20%) = $17,561 > 0, i'% > 20%
PW(25%) = -$41,497 < 0, i'% < 25%
Linear interpolation between 20% and 25% yields: i'% = 21.5% > 10%, so the
new product line appears to be profitable.

111

Chapter 5 Solutions

However, due to the multiple sign changes in the cash flow pattern, the possibility
of multiple IRRs exists. The following graph of PW versus i indicates that
multiple IRRs do not exist for this problem.

PW(i%)
$400,000
$300,000

IRR = 21.5%

$200,000
$100,000
$0
10%

-$100,000

20%

30%

40%

50%

60%

70%

80%

90%

100%

-$200,000
-$300,000
-$400,000

5-38

(a) 15.2% (b) 18.8% (c) 21.5% (d) 20%

5-39

The following cash flow diagram summarizes the known information in this
problem.
F = $54,000

3-25-1993
1-30-2010
i'% = ? per month

P = $13,500
The value of N is the number of time periods separating P and F. If monthly
compounding is assumed, N equals (9 months in 1993) + (192 months from 1994
through 2009) + (1 month in 2010) = 202 months (to the nearest integer).
Therefore, we can determine the unknown interest rate using:

Chapter 5 Solutions

112

F = P (1+ i'%)N; $54,000 = $13,500 (1+ i'%)202


Using logarithms, i'% per month = 0.69%.
The nominal rate of interest = (12)(0.69%) = 8.28% per year and the effective
annual interest rate = [(1.0069)12 - 1](100%) = 8.6% per year.
5-40

(a)

PW(i'%) = 0 = - $23,000 - $1,200 (P/A,i'%,4) - $8,000 (P/F,i'%,4)


+ $5,500 (P/A,i'%,11)(P/F,i'%,4) +$33,000 (P/F,i'%,15)
By linear interpolation, i'% = IRR = 10%

(b)

FW (12%) = - $23,000(F/P,12%,15) - $1,200(F/A,12%,4)(F/P,12%,11)


- $8,000(F/P,12%,11) + $5,500 (F/A,12%,11) + $33,000
= - $23,000(5.4736) - $1,200(4.7793)(3.4785) - $8,000(3.4785)
+ $5,500 (20.6546) + $33,000
= -$27,070.25

(c)

-$23,000 - $1,200(P/A,12%,4) - $8,000(P/F,12%,4) (F/P,i'%,15)


= $5,500(F/A,12%,11) + $33,000
[$23,000 + $1,200(3.0373) + $8,000(0.6355)] (F/P,i'%,15)
= $5,500(20.6546) + $33,000
15
$31,728.76 (1 + i') = $146,600.30
i' = ERR = 0.1074 or 10.74%

5-41

$20,000 = $250 [F/A,i'%,(76-21)] (F/P,i'%,1)


By linear interpolation, i = 1.24 % per year

5-42

PW(15%) = - $1,500 + $200(P/F,15%,1) + $400(P/F,15%,2) + $450(P/F,15%,3)


+ $450(P/F,15%,4) + $600(P/F,15%,5) + $900(P/F,15%,6)
+ $1,100 (P/F,15%,7)
PW(15%) = - $1,500 + $200(0.8696) + $400(0.7561) + $450(0.6575)
+ $450(0.5718) + $600(0.4972) + $900(0.4323) + $1,100(0.3759)
= $630.43 0, therefore the project is acceptable.

113

Chapter 5 Solutions

FW(15%)

= - $1,500 (F/P,15%,7) + $200 (F/P,15%,6) + $400 (F/P,15%,5)


+ $450 (F/P,15%,4) + $450 (F/P,15%,3) + $600 (F/P,15%,2)
+ $900 (F/P,15%,1) + $1,100
FW(15%) = - $1,500 (2.66) + $200 (2.3131) + $400 (2.0114) + $450 (1.749)
+ $450 (1.5209) + $600 (1.3225) + $900 (1.15) + $1,100
= $1,677.14 0, therefore the project is acceptable.
AW(15%) = PW (A/P,15%,7)
= $630.43 (0.2404) = $151.56 0, therefore the project is acceptable.
IRR: PW(i'%) = 0 = - $1,500 + $200 (P/F,i'%,1) + $400 (P/F,i'%,2)
+ $450 (P/F,i'%,3) + $450 (P/F, i'%, 4) + $600 (P/F, i'%, 5)
+ $900 (P/F, i'%, 6) + $1,100 (P/F, i'%, 7)
By linear interpolation, IRR = 24.9% 15%, therefore the project is acceptable.

Simple Payback Period:

(cash flow)

$1,500 when = 4 years < 5 years,

k=1

Therefore, the project is acceptable.


'

Discounted Payback Period:

(cash flow)

(P/F,15%,k) $1,500

k=1

We must find the PW of all cash flows up to year k. Because ' , the
Discounted Payback Period is at least 4 years.
PW0-4(15%) = - $1,500 + $200(P/F,15%,1) + $400(P/F,15%,2)
+ $450(P/F,15%,3) + $450 (P/F,15%,4)
= - $1,500 + $200(0.8696) + $400(0.7561) + $450(0.6575)
+ $450(0.5718)
= -$470.46 < 0 thus ' > 4 years
PW0-5(15%) = -$470.46 + $600 (P/F,15%,5) = -$470.46 + $600 (0.4972)
= -$172.14 < 0 thus ' > 5 years
PW0-6(15%) = -$172.14 + $900 (P/F,15%,6) = -$172.14 + $900 (0.4323)
= $216.93 > 0 thus ' = 6 years 6 years
Therefore the project is acceptable.
ERR: -$1,500(F/P,i'%,7)= $200 (F/P,15%,6) + $400 (F/P,15%,5)
+ $450(F/P,15%,4) + $450 (F/P,15%,3)

Chapter 5 Solutions

114

+ $600 (F/P,15%,2) + $900 (F/P,15%,1) + $1,100


$1,500(F/P,i'%,7) = $200(2.3131) + $400(2.0114) + $450(1.749)
+ $450(1.5209) + $600(1.3225) + $900(1.15) + $1,100
7
i' = 0.209 or 20.9%
$1,500(1 + i') = $5,667.14
Since ERR = 20.9% 15%, the project is acceptable.
5-43

(a)

(b)

5-44

= 4 years thus ' 4 years


PW0-4(25%) = -$500 - $200(P/F,25%,1) + $500(P/F,25%,3)
+ $500(P/F,25%,4)
= -$199.2 < 0, thus ' > 4 years
PW0-5 (25%) = -$199.2 + $500 (P/F,25%,5)
= -$35.35 < 0, thus ' > 5 years
PW0-6(25%) = -$35.35 + $500 (P/F,25%,6)
= $95.70 > 0, thus ' = 6 years
0 = -$500 - $200 (P/F,i'%,1) + $500 (P/A,i'%,4)(P/F,i'%,2)
By linear interpolation, i'% = 29.4% per year

(a) PW(12%) = -P + $90,000 (P/A, 12%, N) + $7,000 (P/F, 12%, N) = 0

Affordable Price (P)

N years
5
6
7
8
9
10

P (= affordable price)
$328,403.80
$373,572.20
$413,908.10
$449,911.30
$482,062.20
$510,772.00

$550,000
$500,000
$450,000
$400,000
$350,000
$300,000
5

5-45

7
8
Useful Life (N)

10

(b)

$344,000 / $90,000 4 years

(a)

PW(15%) = -$100 - $50 (P/F, 15%, 1) + [$20 (P/A, 15%, 4)


+ $100 (P/G, 15%, 4)](P/F, 15%, 2)
= -$100 - $50(0.8696) + [$20(2.8550) + $100 (3.786)](0.7561)

115

Chapter 5 Solutions

= $185.95 0
Yes, this project is financially profitable.
EOY
0
1
2
3
4
5

(b)

Cash Flow
- $100
- 50
0
20
120
220

Balance
- $100
- 150
- 150
- 130
- 10
210

Balance becomes positive at the end of year 5. Thus, = 5 years.


(c)

Since = 5 years, ' 5 years


PW0-5(15%) = -$100 - $50(P/F,15%,1) + [$20 (P/A,15%,3)
+$100 (P/G,15%,3)] (P/F,15%,2)
PW0-5(15%) = -$100 - $50(0.8696) + [$20(2.2832) + $100(2.071)](0.7561)
= $47.63 > 0, thus ' = 5 years

5-46

(a)

PW(i'%) = 0 = -$100,000 + $20,000 (P/A,i'%,5) + $10,000 (P/G,i'%,5)


+ $10,000 (P/F,i'%,5)
PW(20%) = $12,891 > 0, i'% > 20%
PW(25%) = -$897 < 0, i'% < 25%
By linear interpolation, i'% = IRR = 24.7%

(b)

(c)

5-47

EOY
1
2
3
4

Cumulative Cash Flow


-$100,000 + $20,000 = -$80,000 < 0
-80,000 + 30,000 = -50,000 < 0
-50,000 + 40,000 = -10,000 < 0
-10,000 + 50,000 = 40,000 > 0 = 4 years

Although this project is profitable (IRR > MARR), it is not acceptable since
= 4 years is greater than the maximum allowable simple payback period of
3 years.

IRR method:
PW(i'%) = 0 = $500,000(P/F,i'%,1) + $300,000(P/F,i'%,2)
+ [$100,000 + $100,000(P/A,i'%,7) + $50,000(P/G,i'%,7)](P/F,i'%,3)
- $2,500,000 (P/F,i'%,4)

Chapter 5 Solutions

116

i'%
1
2
3
4
5

Present Worth
$103,331.55
63,694.68
30,228.14
2,175.18
-21,130.28

i'%
30
31
32

Present Worth
-$12,186.78
-5,479.09
1,182.76

There are two internal rates of return: 4.9% and 31.2% per year.
ERR Method:
-$2,400,000(P/F,8%,4)(F/P,i'%,10) = $500,000(F/P,8%,9)
+$300,000(F/P,8%,8)
+ $100,000(F/P,8%,7)
+ $150,000(P/A,8%,6)(F/P,8%,6)
+ $50,000(P/G,8%,6)(F/P,8%,6)
After solving, the external rate of return is 7.6% per year.
5-48 (a)

2 sign changes 0,1 or 2 internal rates of return


PW(i) = $1,000 - $5,000 (1+i)-1 + $6,000 (1+i)-2
Let x = 1/(1+i)
PW(i%)
= 1 5x 6x 2
1,000
From the quadratic formula: x = 1/2 and 1/3
If x = 1/2; 1/2 = 1/(1+i); i = 1.00, or 100% per year
If x = 1/3; 1/3 = 1/(1+i); i = 2.00, or 200% per year

(b)

5-49

-$5,000(P/F,10%,1)(F/P,i'%,2) = $1,000(F/P,10%,2) + $6,000


$5,000(0.9091)(F/P,i'%,2) = $1,000(1.21) + $6,000
i' = 0.259 or 25.9% per year
$4,545.50 (1+i')2 = $7,210

The general equation to find the internal rate of return is:


PW(i'%)= 0 = -$520,000 + $200,000 (P/A, i'%, 10) - $1,500,000 (P/F, i'%, 10)
(a)

i'%

PW(i'%)

i'%

PW(i'%)

0
0.5
1
4
10

- $ 20,000
0
10,250
89,000
131,000

15
20
25
30
40

$113,000
76,000
33,000
- 10,350
- 89,100

i'% = 1/2% and 28.8% per year.

117

Chapter 5 Solutions

$150,000
$100,000
PW(i%)

IRR = 28.8%

$50,000
$0
5%

-$50,000
-$100,000

10%

15%

20%

25%

30%

35%

IRR = 0.5%

i%

(b) Assume = 20% per year.

-$520,000-$1,500,000(P/F,20%,10)(F/P,i'%,10)=$200,000(F/A,20%,10)
[$520,000 + $1,500,000(0.1615)](F/P,i'%,10) = $200,000(25.9587)
i' = 0.2115 or 21.15%
$762,250 (1+i')10 = $5,191,740
ERR > 20%, therefore the project is economically acceptable.
5-50

(a)

In all three cases, IRR = 15.3%. This is true for EOY 0 as a reference point
in time, and also for EOY 4 as a reference point in time.

(b)

PW1(10%) = $137.24 at EOY 0


PW2(10%) = $137.24 at EOY 4
at EOY 0
PW2(10%) = $93.73
PW3(10%) = $686.18 at EOY 4
PW3(10%) = $468.67 at EOY 0
Select (3) to maximize PW(10%). However, the PW(IRR=15.3%) would be
zero for all three situations.

5-51 (a) PW = 16X (P/F,15%,16) + 16Y (P/A, 15%,11) (P/F,15%,4)


2Y (P/G,15%,11) (P/F,15%,4)
= 16X (0.1069) + 16Y (5.2337) (0.5718) 2Y (19.129) (0.5718)
= 1.710X + 26.006Y
(b) AW = PW (A/P,15%,20) = 1.710X + 26.006Y (0.1598)
= 0.2732X + $4.1558Y
5-52

(a) PW = 23Y (P/A,15%,10-2) (P/F,15%,2) 3Y (P/G,15%, 10-2) (P/F,14%,2)

Chapter 5 Solutions

118

+ 5Y (P/A,15%, 17-10) (P/F,15%,10) + 2Y (P/G,15%,17-10) (P/F,15%,10)


PW = 23Y (4.4873) (0.7561) 3Y (12.481) (0.7561) + 5Y (4.1604) (0.2472)
+ 2Y (10.192) (0.2472) = 59.91Y
(b) A = 59.91 Y (A/P,15%,17) = 59.91Y (0.1654) = 9.908Y
5-53

(a)

Let Q = monthly payment at age 65


0.8Q = monthly payment at age 62 (20% penalty for early retirement)
Let i = 0.5% per month (6% per year)
How long does it take for the deferred full payment plan to catch up to the
early retirement plan in terms of the future wealth of your uncle?
0.8Q (F/A,1/2%,N) = Q(F/A,1/2%,N-36)
( F / A,1/ 2%, N - 36)
0.8 =
( F / A,1/ 2%, N)

(Basic Equation)

By trial and error, it takes N 348 months (past age 62) before the
deferred plan overtakes the early plan. So, it takes 29 years past age 62
before Social Security retirement at age 65 is preferred. If your uncle's
health is good and he has longevity in his family tree, he should probably
start drawing social security payments at age 65.
(b)

At 1.5% per month, you can't live long enough (practically speaking) to
catch
up to the early retirement option. So take Social Security starting
at age 62!
The basic calculation is shown below:
F
/
A,1.5%,
N - 36)
(
(Basic Equation)
0.8 =
( F / A,1.5%, N)
By trial and error: age 85, 0.8 > 0.578; age 100, 0.8 > 0.584; and age 120,
0.8 > 0.585.

(c)

At i = 0% per month, the basic equation is:


( F / A,0%, N - 36)
0.8 =
( F / A,0%, N)
By trial and error, at age 77 0.8 0.795, or basic breakeven. But at age 85,
FWearly = $220,800 and FWdeferred = $240,800. Thus, if your uncle expects to
live past age 77, deferring social security payments to the regular age of 65
years is preferred at i = 0% per month.

119

Chapter 5 Solutions

5-54

$10,000,000

2000

2004 2005 2006

2009

2014 ~ Forever

$250,000/yr

X
X
Amount at July 2004:
$100,000(1.05)4 - $3,000,000 = $12,155,000 $3,000,000 = $9,155,000
$9,155,000

2004 2005

2009

2014

$250,000/yr
X

$9,155,000 = $250,000(P/A,5%, ) + X(A/F,5%,5)(P/A,5%, )


= $250,000 (20) + 0.181X (20)
or X = $1,147,790 every 5 years
5-55

(a)

AW(15%) = -$710,000 (A/P,15%,5) + $198,000 + $100,000 (A/F,15%,5)


= $1,828 per year. Yes, its a good investment opportunity.

(b)

IRR: 0 = -$710,000 (A/P, i',5) +$198,000 +$100,000(A/F, i',5)


so i' = 15.3%
= 4 years; ' = 5 years

(c) Other factors include sales price of reworked units, life of the machine, the
companys reputation, and demand for the product.

Chapter 5 Solutions

120

Solutions to Spreadsheet Exercises


5-56
Desired Ending Balance
Current Age
Age at
Retirement

$ 250,000
25
60
Interest Rate per
Year
8%
$ 2,207
$ 3,420
$ 5,463
$ 9,207

4%
N

5
10
15
20

$
$
$
$

4,458
6,003
8,395
12,485

$
$
$
$

12%
1,036
1,875
3,470
6,706

The formula used in cell C7 is:


=-PMT(C$6,$D$3-$D$2-$B7,,$D$1)
Note that this uses the fv parameter of the PMT function instead of the
pv parameter. The formula in cell C7 was copied over the range C7:E10.
The trend in the table shows that as the interest rate increases,
less has to be saved each year. Also, the longer Jane delays the
start of her annual savings, the larger the annual deposit will have to be.

121

Chapter 5 Solutions

5-57
MARR =
Capital
Investment =
Market Value =
Useful Life =
Annual Savings
=
Annual
Expense =
EOY
0
1
2
3
4
5
5

11%
$
$
$

10,000
2,000
5

5,311

3,000

$
$
$
$
$
$
$

Cash Flow
(10,000)
2,311
2,311
2,311
2,311
2,311
2,000

EOY
0
1
2
3
4
5

Present Value =
Annual Worth =
Future Worth =
Internal Rate of
Return =

$
$
$
$
$
$

Cash Flow
(10,000)
2,311
2,311
2,311
2,311
4,311

$
$
$

(271.88)
(73.56)
(458.13)
10.0%

From Example 5-11, the IRR = 10%. Therefore, any MARR > 10% will result in
negative equivalent worth values.
The following table displays the results of different MARRs on the profitability
measures.

Present Worth
Annual Worth
Future Worth

Chapter 5 Solutions

MARR = 15%
MARR = 5%
$
(1,258.82)
$ 1,572.47
$
(375.52)
$ 363.20
$
(2,531.93)
$ 2,006.92

122

5-58
MARR =
Reinvestment rate =
Capital Investment
=
Market Value =
Useful Life =
Annual Savings =
Annual Expense
=
EOY
0
1
2
3
4
5
5

20%
20%
$
$
$
$

25,000
5,000
5
8,000

$
$
$
$
$
$
$

Cash Flow
(25,000)
8,000
8,000
8,000
8,000
8,000
5,000

EOY
0
1
2
3
4
5

Present Value =
Annual Worth =
Future Worth =
Internal Rate of
Return =
External Rate of
Return =

$
$
$
$
$
$

Cash Flow
(25,000)
8,000
8,000
8,000
8,000
13,000

$
$
$

934.28
312.41
2,324.80
21.58%
20.88%

This spreadsheet was created using the spreadsheet for Example 5-11.
The following table displays the results of different MARRs on the profitability
measures.

Present Worth
Annual Worth
Future Worth
IRR
ERR

MARR = 18%
MARR = 22%
$ 2,202.91
$(240.89)
$
704.44
$ (84.12)
$ 5,039.73
$(651.04)
21.58%
21.58%
20.88%
20.88%

The original recommendation is unchanged for a MARR = 18%. However, the recommendation
does change for MARR = 22% (which is greater than the IRR of the project's cash flows).
Note that the ERR is unaffected by changes in the MARR. This is because 1) the reinvestment
rate was assumed to remain at 20%, and 2) there is only a single net cash outflow occurring at t=0.

123

Chapter 5 Solutions

5-59
MARR =
Capital Investment
=
Market Value =
Useful Life =
Net Annual
Savings =
EOY
0
1
2
3
4
5
5

20%
$
$
$

28,750
5,000
5

8,000

$
$
$
$
$
$
$

Cash Flow
(28,750)
8,000
8,000
8,000
8,000
8,000
5,000

EOY
0
1
2
3
4
5

$
$
$
$
$
$

Cash Flow
(28,750)
8,000
8,000
8,000
8,000
13,000

IRR =
The following IRR values were determined by changing the values of
the capital investment (cell B2) and market value (cell (B3) in the
spreadsheet.
Capital Investment
= (1+0.15)*25000
= (1-0.15)*25000

IRR
15.76%
29.03%

Market Value
= (1+0.15)*5000
= (1-0.15)*5000

IRR
22.06%
21.08%

A change in the capital investment amount has a larger impact on IRR.


This is a result of both the magnitude of the cash flow and its timing.

Chapter 5 Solutions

124

15.76%

Solutions to Case Study Exercises


5-60

Average number of wafers per week:


(10 wafers/hr)(168 hr/wk)(0.90) = 1,512
Added profit per month:
(1,512 wafers/wk)(4.333 wk/month)($150/wafer) = $982,724
PW(1%) = -$250,000 - $25,000(P/A, 1%, 60) + $982,724(P/A, 1%, 60) =
$42,804,482
The increase in CVD utilization serves to make the project even more attractive.

5-61

Average number of wafers per week:


(15 wafers/hr)(168 hr/wk)(0.80) = 2,016
New breakeven point:
X=

$1,373,875
= $3.50 / wafer
(2,016)(4.333)(44.955)

$3.50/$100 = 0.035 extra microprocessors per wafer.


The retrofitted CVD tool would significantly reduce the breakeven point.
5-62

Average number of wafers per week:


(10 wafers/hr)(150 hr/wk)(0.90) = 1,350
Added profit per month:
(1,350 wafers/wk)(4.333 wk/month)($150/wafer) = $877,433
PW(1%) = -$250,000 - $25,000(P/A, 1%, 60) + $877,433(P/A, 1%, 60)
= $38,071,126
New breakeven point:
X=

$1,373,875
= $5.225 / wafer
(1,350)(4.333)(44.955)

$5.225/$100 = 0.05225 extra microprocessors per wafer.

125

Chapter 5 Solutions

Solutions to FE Practice Problems


5-63

i / mo. = 9%/12 = % per month; N = 4 x 12 = 48 months


A = $8,000 (A/P, % per month, 48 months)
= $8,000 (.0249) = $199.20
Select (d)

5-64

EOY
0
1
2
3
4

Cumulative PW (i = 12%)
-$300,000
-$300,000 + $111,837.50(P/F,12%,1) = -$200,140.30
-200,140.30 + $111,837.50(P/F,12%,2) = -$110,983.45
-$110,983.45 + $111,837.50(P/F,12%,3) = -$31,377.52
-$31,377.52 + $111,837.50(P/F,12%,4) = $39,695.21 > 0
' = 4

Select (a)
5-65

$14,316 = X(A/P,8%, )
$14,316 = X (0.08); X = $178,950
Select (a)

5-66

Find i% such that PW(i%) = 0


0 = -$3,345 + $1,100(P/A,i'%,4)
PW(10%) = $141.89 tells us that i'% > 10%
PW (12%) = -$3.97 tells us that i'% < 12% (but close!)

IRR = 11.95%
Select (b)

5-67

VN = C(P/F,i%,N) + rZ(P/A,i%,N) = $981


N = 8 periods
r = 10% per period ($1,000/$100 = 10%)
C = Z = $1,000
$981= $1,000(P/F,i%,8) + (.10)($1,000)(P/A,i%,8)
$981 = $1,000(P/F,i%,8) + $100(P/A,i%,8)
try i = 10%; $466.50 + $533.49 = $999.99
try i = 12%; $403.90 + $496.76 = $900.66
by observation, i% is > 10% but very close to 10%
rate of return = 10.35%

Chapter 5 Solutions

126

Select (c)
5-68

AW = -$5,123 + $1,100 (P/A,10%,20)


(P/A,10%,20) = 4.6573
Using the interest tables, (P/A,10%,6) = 4.3553 and (P/A,10%,7) = 4.8684.
Thus, 6 < N < 7. Using linear interpolation we find that N 6.5.
N = ' = 7
Select (a)

5-69 AW = ($5,000 + $500 (A/G.1%24))*(F/P,1%,12)


= ($5,000 + $500*11.02337)*1.0100*12.6825
= $134,649

Select (e)
5-70

i = 12%

f = 5%
$7000[1 (P/F, 12%, 6)(F/P, 5%, 6)]
0.12 0.05
$7000[1 (0.5066)(1.3401)]
= $9,000 +
0.07
= $41,110.53
AW(12%) = $41,110.53 (A/P, 12%, 6) = $41,110.53(0.2432)
= $9,998.08
PW (12%) = $9,000 +

Select (c)

5-71

For simplicity, assume a loan amount of $1,000. The yearly payment due on this
loan is
$1,000 (A/P, 6%, 5) = $1,000 (0.2374) = $237.40.
The application fee due is $1,000 (0.1367) = $136.70. Thus the borrower walks
away with $1,000 - $136.70 = $863.30. The effective interest rate being charged
can be found by solving the following equivalence equation for i':
$863.30 = $237.40 (P/A,i'%,5)
(P/A,i'%,5) = 3.6365
Using the interest tables, (P/A,10%,5) = 3.7908 and (P/A,12%,5) = 3.6048. Thus,
127

Chapter 5 Solutions

10% < i'% < 12%. Using linear interpolation we find that i' = 11.65%.
Select (a)
5-72

P = $8,000 (P/A,12%,4)(P/F,12%,16)
= $8,000 (3.0373) (0.1631) = $3,963.07
Select (d)

Chapter 5 Solutions

128

Vous aimerez peut-être aussi